Amperesches Gesetz und Satz von Stokes für die Stromdichte

Sowohl bei Griffiths als auch bei Jackson wird das Amperesche Gesetz (oder umgekehrt die Drehung des Magnetfelds) durch Anwendung des Satzes von Stokes auf das Oberflächenintegral der Stromdichte J abgeleitet. Das Argument beruht auf der Tatsache, dass

ICH e N C = S J D A
aber ich habe Mühe zu verstehen, wie dies im Allgemeinen für jede Stromdichte und jede begrenzte Oberfläche gelten könnte S .

Natürlich gilt dies für einen geraden Draht, der durch die Mitte einer Amperschen Schleife verläuft, wenn der Vektor J parallel zum Normalenvektor der einfachsten von der Schleife eingeschlossenen Oberfläche ist, aber was ist, wenn der Draht in Bezug auf die leicht abgewinkelt ist Ebene? Unter Verwendung der gleichen ebenen Fläche wird nun der Strom umschlossen

S ICH δ ( X ) [ X j z ] N ^ D A
was eindeutig nicht der gleiche Wert ist. Berücksichtigt das Amperesche Gesetz also nur das Magnetfeld, das von Vektoren parallel zur Normalen erzeugt wird?

Und meine zweite Frage ist diese. Wie gilt der Satz von Stokes im zweiten Fall, wo die Kräuselung nur entlang einer einzelnen Linie ungleich Null ist? Würde eine andere Oberfläche, die sich in eine dritte Dimension erstreckt und eine Normale parallel zum Strom am Schnittpunkt hat, nicht einen anderen Wert für das Oberflächenintegral ergeben, da die tatsächliche Fläche der Oberfläche keine Rolle spielt? Offensichtlich gibt es einen ernsthaften Fehler in meinem mathematischen Verständnis des Materials, aber es ist für mich nicht offensichtlich.

Danke

Antworten (1)

Ihre erste Gleichung, die nur eine Definition für den Strom ist, gilt für reguläre Vektorfelder J . Es ist nicht einfach, es auf Dirac-Delta-Funktionen anzuwenden. Wenn Sie einen Draht endlicher Größe betrachten, wenn er bzgl S der Schnittpunkt ist um einen Faktor größer 1 / cos θ das kompensiert genau das cos θ Faktor, der sich aus dem Skalarprodukt ergibt.

Nun zu Ihrer zweiten Frage. Beginnen wir damit, die Stromdichte für einen unendlich dünnen Draht richtig zu schreiben, der entlang der gelegt wird z Achse (sie wird durch Gleichungen definiert X = 0 , j = 0 , daher die beiden Delta-Funktionen):

J = ICH δ ( X ) δ ( j ) z ^ .
Nehmen wir die Integrationsfläche an S ist ein Rechteck in einer Ebene ( X ' , j ) das ist um einen Winkel geneigt θ in Bezug auf die ( X , j ) Ebene. Von der Seite gesehen:

Seitenansicht, in der (x,z)-Ebene

Das Normale N (entlang a z ' Achse abgewinkelt durch θ wrt z ) hat Koordinaten ( Sünde θ , 0 , cos θ ) im ( X , j , z ) System. So

S J · N D S = S ICH δ ( X ) δ ( j ) cos θ D S .

Dann müssen wir die Delta-Funktionen in ausdrücken X ' j Koordinaten. j ist unverändert. Gleichung X = 0 wird X ' cos θ + z ' Sünde θ = 0 , So δ ( X ) wird ersetzt durch δ ( X ' cos θ + z ' Sünde θ ) . An S , z ' ist immer null, also müssen wir endlich rechnen

S ICH δ ( X ' cos θ ) δ ( j ) cos θ D S .
Nun, Sie sollten es wissen δ ( k X ' ) = 1 | k | δ ( X ' ) , somit
S ICH δ ( X ' cos θ ) δ ( j ) cos θ D S = S ICH δ ( X ' ) δ ( j ) D S = ICH .
(Alternativ ausdrücken D S = D X ' D j und Ersatzvariable X = X ' cos θ .)

Danke. Das ist sehr hilfreich. Ich habe nicht viel formale Ausbildung in Delta-Funktionen - könnten Sie mich in diesem Fall in die richtige Richtung weisen? Meine zweite Frage ist wirklich nur eine Wiederholung der ersten. Wenn Sie einen einzelnen, infinitesimal kleinen Draht haben, wie kann der Wert des Integrals über eine beliebige Fläche begrenzt werden? S gleich sein? Kann der Normalenvektor im Schnittpunkt je nach Oberfläche nicht buchstäblich in jede Richtung zeigen? Wie kann
S J N D S
für jedes S konstant sein, wenn J eine Delta-Funktion ist und n in jede Richtung zeigen kann?
Mit anderen Worten, wenn die Locke von B, μ J , ist überall nullwertig, außer entlang eines einzelnen infinitesimal kleinen (geraden) Drahtes, wo die Größe und Richtung von J festgelegt ist, wie kann das Oberflächenintegral über jede begrenzte Oberfläche sein S mit allen möglichen normalen gleich sein? Wenn J fest ist und n ein beliebiger Vektor sein kann, wie gilt dann der Satz von Stokes? Danke!